Discover Radius of Convergent for the Series: (1-2^n)(ln(n))x^n - Solved!

  • Thread starter Thread starter danni7070
  • Start date Start date
  • Tags Tags
    Convergent Radius
Click For Summary
The discussion focuses on finding the radius of convergence for the series ∑(1-2^n)(ln(n))x^n. The limit of the ratio of consecutive terms is evaluated using the formula 1/R = L, leading to the conclusion that R = 1/2. Participants clarify the cancellation of logarithmic terms in the limit as n approaches infinity, noting that ln(n)/ln(n+1) approaches 1. The conversation highlights the importance of rigor in mathematical proofs, with suggestions to apply L'Hôpital's rule for clarity. Ultimately, the solution is confirmed as correct, emphasizing the collaborative nature of problem-solving in mathematics.
danni7070
Messages
91
Reaction score
0
[Solved] Radius of Convergent

Homework Statement



Find the radius of convergent for \sum_{n=1}^\infty (1-2^n)(ln(n))x^n



Homework Equations



\frac {1}{R} = L = \lim \frac{a_{n+1}}{a_n}


The Attempt at a Solution



lim \frac {(1-2^{n+1})(ln(n+1)}{(1-2^n)(ln(n))} = L

lim \frac {(1-2^n)(ln(n))}{(1-2^{n+1})(ln(n+1))} = R

I'm dizzy looking at this but how can I find:

\lim_{n\rightarrow\infty} R
 
Last edited:
Physics news on Phys.org
my idea would be to take the quotient of the logs as 1 because the derivatives both go as 1/x and so both functions should behave the same in infinity.
The ones up front can be thrown away don´t make any contribution at infinity so you´re left with 2^n/2^n+1 = 1/2 = R :)

No make it a bit more rigorous if you like :)
 
lim\frac{1-2^n}{1-2^{n+1}} * \frac{ln(n)}{ln(n+1)} = \frac{2^n}{2^{n+1}} = 2^{n-(n+1)} = 2^{-1} = \frac{1}{2}

Indeed this is right, I've checked the results and it is a hit!

But I don't underssand why you can just skip the log parts?

Ahh, now when I think about I see that lim \frac{ln(n)}{ln(n+1)} = 1 so that cancels out!

I usually get stuck with the obvious.

Thanks mr. Brown!
 
you could use l´hospital on the logs to make it rigorous be the derivative quotient would be
n^-1/(n+1)^(-1)=1+1/n goes to n as n goes to infinity :)
 
Yeah I know but this lim is just so much for the eye to solve :)
 
Question: A clock's minute hand has length 4 and its hour hand has length 3. What is the distance between the tips at the moment when it is increasing most rapidly?(Putnam Exam Question) Answer: Making assumption that both the hands moves at constant angular velocities, the answer is ## \sqrt{7} .## But don't you think this assumption is somewhat doubtful and wrong?

Similar threads

  • · Replies 3 ·
Replies
3
Views
1K
  • · Replies 2 ·
Replies
2
Views
2K
  • · Replies 2 ·
Replies
2
Views
2K
Replies
5
Views
2K
  • · Replies 1 ·
Replies
1
Views
2K
  • · Replies 7 ·
Replies
7
Views
2K
  • · Replies 5 ·
Replies
5
Views
2K
  • · Replies 34 ·
2
Replies
34
Views
3K
  • · Replies 4 ·
Replies
4
Views
1K
  • · Replies 8 ·
Replies
8
Views
2K